% https://tex.stackexchange.com/q/369803 \documentclass[a4paper,parskip=half]{scrartcl} \usepackage[utf8]{inputenc} \usepackage[ngerman]{babel} \usepackage{amsmath} \usepackage{amsthm} \usepackage{amsfonts} \usepackage{amssymb} \usepackage[left=2cm,right=2.5cm,top=2.5cm,bottom=2cm]{geometry} \usepackage{xsim,siunitx} \DeclareExerciseTagging{difficulty} \DeclareExerciseEnvironmentTemplate{custom} {% \subsection* {% \XSIMmixedcase{\GetExerciseName}\nobreakspace \GetExerciseProperty{counter}% \IfInsideSolutionF {% \IfExercisePropertySetT{subtitle} { {\normalfont(\GetExerciseProperty{subtitle})}}% }% }% }{} \xsimsetup{ exercise/name = \XSIMtranslate{question} , exercise/template=custom , solution/print=true } \begin{document} \begin{exercise}[ID=wdsw, subtitle = Widerstandswürfel , difficulty = 2] Gegeben ist ein Würfel, wobei jede der Kanten einen Widerstand von $R = \SI{1}{\ohm}$ hat. Wie groß ist der Widerstand entlang einer Raumdiagonale? \end{exercise} \begin{solution} Wir wollen den Widerstand zwischen den Punkten $X$ und $Y$ bestimmen, also entlang der Raumdiagonale (siehe Abb. \ref{fig:wdsws1}). Weil die Raumdiagonale eine Symmetrieachse ist, sollte das Problem symmetrisch sein, und deswegen eine recht einfache Lösung haben. \end{solution} \end{document}